Weaken question - trap answer choice or no?

blackhawksfanblackhawksfan Alum Member
edited May 2014 in Logical Reasoning 55 karma
Hi fellow 7sagers,
As a review tonight I decided to write down steps I take for weakening questions. I’d appreciate any response to this question I have... PLEASE! It’s been bothering me for the past 2 hours. I realize that this might require going to the videos that I refer to below – so I thank you in advance for your time spent!

For Weaken Qs, we are taught to attack the premise-conclusion relationship; that is, the support for our conclusion. So I tried to come up with an example:

If the Premise/Conclusion is: TV sales increase, because Survey A indicates so.
A trap answer choice would be ‘Survey B indicates otherwise’ (right?) because sure, Survey B is a contradiction, but our premise about Survey A still holds true and we can’t doubt its validity since it was given to us.

This is corroborated by a video explanation I watched, PT 60 Section 1 Question 13 (in short – there’s an answer choice (B) saying Survey X says some dangerously out of scope stuff – but our premise is about Survey Y saying whatever to support the conclusion.)

BUT I was watching the “Serious Medical Condition – Weaken Question” video lesson and in that question, Answer Choice (A) serves as a perfectly acceptable weaken-er! But it is another one of those ‘in another study....this was shown...” !!

So does that type of answer choice weaken the argument? Help appreciated!! Thank you :)

Comments

  • NellysLSATNellysLSAT Alum Member
    edited May 2014 186 karma
    that question (serious medical condition) is an Except question.
    A does weaken the argument along with four other choices amongst the group.
    And C is the only one that doesn't weaken the argument or does anything much for the argument.
    did I understand your question correctly?
  • blackhawksfanblackhawksfan Alum Member
    55 karma
    Hi there! Thanks for responding. Yes- you are right, answer choice (a) in the medical example does indeed weaken. I am not confused about that particular question but rather, why in that case, 'another study' is acceptable as a weakener, yet in the other example I mentioned, 'another study' is out of scope.

    I hope I clarified it better!
  • NellysLSATNellysLSAT Alum Member
    186 karma
    ah I get it now. sorry
    Ill have to wait for someone else to help. as I haven't taken prep test 60 and don't want to look at it as yet.
  • Jonathan WangJonathan Wang Yearly Sage
    6866 karma
    Using another study isn't blanket right or wrong; you have to evaluate it in the context of the argument it's being put into. Re-read the conclusions you're trying to draw in each case and think about how they differ; this should put you on the right path for figuring out why the survey weakens in one case but not the other.
  • blackhawksfanblackhawksfan Alum Member
    55 karma
    Hi Jonathan, thanks for your input.

    (Warning: partial PT 60 question SPOILER alert below)

    I re-read the conclusions for both questions. In the PT 60 example, I can see that the conclusion is supported by the surveys... but it's a terrible support because the economists make a different claim (they made a claim about the strongest incentive, NOT preference). In the medical condition video lesson, the conclusion is also supported by the studies... but there isn't another claim to compare it to.

    Still quite lost. So what I'm getting is that there's no black/white rule about using another survey/study to weaken answer choices and that it depends on the context of the argument?
  • Jonathan WangJonathan Wang Yearly Sage
    edited May 2014 6866 karma
    Exactly. The tactic of bringing up clashing survey results obviously has its place, but only in certain discussions. Whether or not something is a flaw is completely reliant on the thing you're trying to prove and its relationship to the information that you have. Recognizing a potentially flawed method of reasoning is only step 1 - you still need to figure out whether it's actually a problem in the given case.
  • blackhawksfanblackhawksfan Alum Member
    55 karma
    Gotcha- thank you Jonathan!
Sign In or Register to comment.